To solve the problem, use the given probabilities and conditional probability to find \(N\).
Given:
- Total balls = \(N\)
- White balls = 3
- Green balls = 6
- Blue balls = \(N - 9\) (since 3 + 6 = 9)
- Events:
\(W_i\) = \(i^{th}\) ball drawn is white
\(G_i\) = \(i^{th}\) ball drawn is green
\(B_i\) = \(i^{th}\) ball drawn is blue
- \(P(W_1 \cap G_2 \cap B_3) = \frac{2}{5N}\)
- \(P(B_3 | W_1 \cap G_2) = \frac{2}{9}\)
Step 1: Write expression for \(P(W_1 \cap G_2 \cap B_3)\)
Since drawing is without replacement:
\[
P(W_1 \cap G_2 \cap B_3) = P(W_1) \times P(G_2 | W_1) \times P(B_3 | W_1 \cap G_2)
\]
Calculate each term:
\[
P(W_1) = \frac{3}{N}
\]
After drawing one white ball, remaining balls = \(N-1\), green balls = 6:
\[
P(G_2 | W_1) = \frac{6}{N - 1}
\]
Given:
\[
P(B_3 | W_1 \cap G_2) = \frac{2}{9}
\]
So,
\[
P(W_1 \cap G_2 \cap B_3) = \frac{3}{N} \times \frac{6}{N-1} \times \frac{2}{9} = \frac{36}{9 N (N-1)} = \frac{4}{N (N-1)}
\]
Step 2: Equate this to given value \(\frac{2}{5 N}\):
\[
\frac{4}{N (N - 1)} = \frac{2}{5 N}
\]
Multiply both sides by \(N (N-1) 5 N\):
\[
4 \times 5 N = 2 \times (N)(N - 1)
\]
\[
20 N = 2 N (N - 1)
\]
Divide both sides by \(2 N\) (assuming \(N \neq 0\)):
\[
10 = N - 1
\]
\[
N = 11
\]
Final Answer:
\[
\boxed{11}
\]
To solve the problem, we need to find the value of $N$, the total number of balls in the collection.
1. Given Data:
We are given that $W = 3$, $G = 6$, and $B = N - 9$ where $N$ is the total number of balls.
2. Probability Calculation for $W_1 \cap G_2 \cap B_3$:
The probability of drawing a white ball first, a green ball second, and a blue ball third is given by:
$ P(W_1 \cap G_2 \cap B_3) = P(W_1) P(G_2 | W_1) P(B_3 | W_1 \cap G_2) $
Substituting the values, we get:
$ P(W_1 \cap G_2 \cap B_3) = \frac{3}{N} \times \frac{6}{N-1} \times \frac{N-9}{N-2}$.
3. Setting Up the Equation:
We are given that $ P(W_1 \cap G_2 \cap B_3) = \frac{2}{5N} $, so we equate the two expressions:
$ \frac{3}{N} \times \frac{6}{N-1} \times \frac{N-9}{N-2} = \frac{2}{5N}$.
4. Solving for $N$:
Multiply both sides of the equation to get:
$ \frac{18(N-9)}{N(N-1)(N-2)} = \frac{2}{5N}$.
Now, multiply both sides by $5N$:
$ 9(N-9) \cdot 5N = N(N-1)(N-2)$.
5. Simplifying the Equation:
Expanding the terms, we get:
$ 45(N-9) = N^2 - 3N + 2$.
Now simplify further:
$ 45N - 405 = N^2 - 3N + 2$.
Rearranging the terms, we get:
$ N^2 - 48N + 407 = 0$.
6. Verifying $P(B_3 | W_1 \cap G_2)$:
Next, we are given that $ P(B_3 | W_1 \cap G_2) = \frac{2}{9}$. We calculate $ P(W_1 \cap G_2)$ as:
$ P(W_1 \cap G_2) = P(W_1) P(G_2 | W_1) = \frac{3}{N} \times \frac{6}{N-1} = \frac{18}{N(N-1)}$.
Substituting into the equation for $P(B_3 | W_1 \cap G_2)$, we get:
$ P(B_3 | W_1 \cap G_2) = \frac{\frac{3}{N} \times \frac{6}{N-1} \times \frac{N-9}{N-2}}{\frac{3}{N} \times \frac{6}{N-1}} = \frac{N-9}{N-2}$.
We are given that $ P(B_3 | W_1 \cap G_2) = \frac{2}{9}$, so:
$ 9(N-9) = 2(N-2)$.
Expanding both sides gives:
$ 9N - 81 = 2N - 4$.
Solving for $N$ gives:
$ 7N = 77$ and $N = 11$.
7. Final Verification:
Substituting $N = 11$ into the equation for $P(W_1 \cap G_2 \cap B_3)$:
$ P(W_1 \cap G_2 \cap B_3) = \frac{3}{11} \times \frac{6}{10} \times \frac{2}{9} = \frac{36}{990} = \frac{18}{495} = \frac{2}{55}$.
Since $P(W_1 \cap G_2 \cap B_3) = \frac{2}{5N}$, we verify that $ \frac{2}{55} = \frac{2}{5N}$, so $N = 11$ is correct.
Final Answer:
The final answer is $\boxed{11}$.
A shop selling electronic items sells smartphones of only three reputed companies A, B, and C because chances of their manufacturing a defective smartphone are only 5%, 4%, and 2% respectively. In his inventory, he has 25% smartphones from company A, 35% smartphones from company B, and 40% smartphones from company C.
A person buys a smartphone from this shop
(i) Find the probability that it was defective.
Let $ a_0, a_1, ..., a_{23} $ be real numbers such that $$ \left(1 + \frac{2}{5}x \right)^{23} = \sum_{i=0}^{23} a_i x^i $$ for every real number $ x $. Let $ a_r $ be the largest among the numbers $ a_j $ for $ 0 \leq j \leq 23 $. Then the value of $ r $ is ________.
Let $ \mathbb{R} $ denote the set of all real numbers. Then the area of the region $$ \left\{ (x, y) \in \mathbb{R} \times \mathbb{R} : x > 0, y > \frac{1}{x},\ 5x - 4y - 1 > 0,\ 4x + 4y - 17 < 0 \right\} $$ is
The center of a disk of radius $ r $ and mass $ m $ is attached to a spring of spring constant $ k $, inside a ring of radius $ R>r $ as shown in the figure. The other end of the spring is attached on the periphery of the ring. Both the ring and the disk are in the same vertical plane. The disk can only roll along the inside periphery of the ring, without slipping. The spring can only be stretched or compressed along the periphery of the ring, following Hooke’s law. In equilibrium, the disk is at the bottom of the ring. Assuming small displacement of the disc, the time period of oscillation of center of mass of the disk is written as $ T = \frac{2\pi}{\omega} $. The correct expression for $ \omega $ is ( $ g $ is the acceleration due to gravity): 